497 lines
12 KiB
TeX
497 lines
12 KiB
TeX
\ifextras\else
|
|
\section{Simple problems}
|
|
\fi
|
|
|
|
|
|
% Sherlock, A little exercise
|
|
\problem{A little exercise}
|
|
%\difficulty{1}{5}
|
|
\onestars{3}
|
|
Black has just moved in the game below. White started on the south side of the board.\par
|
|
What was Black's last move, and what was White's last move? \par
|
|
|
|
\ifextras\else
|
|
\note[Note]{
|
|
The boards below are identical copies. Scribble to your heart's content.\\
|
|
There a few empty boards at the end of this handout as well.
|
|
}
|
|
\fi
|
|
|
|
\manyboards{
|
|
ka8,Kc8,
|
|
Ph2,
|
|
Bg1
|
|
}
|
|
|
|
\begin{hintlist}
|
|
Part 1: \tab\threestars{0}{1}{2} \par
|
|
\hintcontent{
|
|
What was Black's last move? What did White do to make this happen?
|
|
}
|
|
|
|
Part 2: \tab\threestars{1}{1}{1} \par
|
|
\hintcontent{
|
|
White uncovered a check. What piece did Black capture?
|
|
}
|
|
|
|
\vspace{2mm}
|
|
Done: \tab\threestars{2}{1}{0}
|
|
\end{hintlist}
|
|
|
|
\begin{solution}
|
|
It's pretty clear that Black just moved out of check from A7.
|
|
|
|
\vspace{2mm}
|
|
|
|
How did White deliver this check? The bishop couldn't have moved to G1,
|
|
so this check must have been discovered by another piece. Since there are
|
|
no extra pieces on the board, Black must've captured this piece on his last move.
|
|
|
|
\vspace{2mm}
|
|
|
|
The only piece that could have moved from the white bishop's diagonal to
|
|
then be captured on A8 is a knight.
|
|
|
|
\vspace{2mm}
|
|
|
|
\textbf{Note:}
|
|
There are two possible answers if we don't know who started where.
|
|
If Black had started on the south side of the board, that bishop could be a promoted pawn.
|
|
\end{solution}
|
|
|
|
\vfill
|
|
\pagebreak
|
|
|
|
|
|
|
|
|
|
|
|
|
|
|
|
|
|
|
|
|
|
% Sherlock, Which color?
|
|
\problem{Which color?}
|
|
%\difficulty{2}{5}
|
|
\onestars{4}
|
|
In the game below, no pieces have moved from a black square to a white square, or from a white square to a black square.
|
|
There is a pawn at G3. What color is it? \par
|
|
As before, White started on the bottom of the board.
|
|
|
|
\manyboards{
|
|
ke8,
|
|
Kb4,
|
|
Ug3,
|
|
Pd2,Pf2
|
|
}
|
|
|
|
|
|
\begin{hintlist}
|
|
Part 1: \tab\threestars{0}{1}{3} \par
|
|
\hintcontent{
|
|
How did the white king get off E1? It must have castled!
|
|
}
|
|
|
|
Part 2: \tab\threestars{1}{1}{2} \par
|
|
\hintcontent{
|
|
It castled kingside (how do we know?) \par
|
|
Now, how did it get off G1?
|
|
}
|
|
|
|
\vspace{2mm}
|
|
Done: \tab\threestars{2}{2}{0}
|
|
\end{hintlist}
|
|
|
|
\begin{solution}
|
|
The white king is the key to this solution. How did it get off of E1? \par
|
|
It must have castled kingside---castling queenside would move a rook from black to white.
|
|
|
|
\vspace{2mm}
|
|
|
|
Now, the white king is on G1. How did it get out of there? \par
|
|
It's must have moved through H2 and G3, which would be impossible if the mystery pawn on G3 was white.
|
|
Therefore, that pawn must be black.
|
|
\end{solution}
|
|
|
|
\vfill
|
|
\pagebreak
|
|
|
|
|
|
|
|
|
|
|
|
|
|
|
|
|
|
|
|
|
|
|
|
|
|
|
|
% Arabian Knights 2
|
|
\problem{Invisible, but not invincible}
|
|
%\difficulty{2}{5}
|
|
\onestars{4}
|
|
|
|
The black king has turned himself invisible. Unfortunately, his position is hopeless. \par
|
|
Mate the king in one move. \par
|
|
|
|
\manyboards{
|
|
Ra8,rb8,Kf8,
|
|
Nb7,Pc7,
|
|
Pa6,Rc6
|
|
}
|
|
|
|
\begin{hintlist}
|
|
Part 1: \tab\threestars{0}{1}{3} \par
|
|
\hintcontent{Don't forget about promotion.}
|
|
|
|
\vspace{2mm}
|
|
Done: \tab\threestars{1}{3}{0}
|
|
\end{hintlist}
|
|
|
|
\begin{solution}
|
|
Since it is White's move, Black cannot be in check. \par
|
|
So, either White is in check or the black king is on C8. \par
|
|
If White is in check, Black must have administered this check by moving from C8 to D7. \par
|
|
Therefore, the black king must be on C8 or D7.
|
|
|
|
\vspace{2mm}
|
|
|
|
If we capture the black rook on B8 with the pawn on C7 and promote it to a knight, the black king will be in checkmate
|
|
regardless of his position.
|
|
\end{solution}
|
|
|
|
\vfill
|
|
\pagebreak
|
|
|
|
|
|
|
|
|
|
|
|
|
|
% Sherlock, a question of survival
|
|
\problem{An empty board}
|
|
%\difficulty{2}{5}
|
|
\onestars{4}
|
|
In the game below, no pieces have moved from a black square to a white square, or from a white square to a black square.
|
|
There is one more piece on the board, which isn't shown. What color square does it stand on? \par
|
|
|
|
\manyboards{
|
|
ke8,
|
|
Pd2,Pf2,
|
|
Ke1
|
|
}
|
|
|
|
|
|
\begin{hintlist}
|
|
Part 1: \tab\threestars{0}{1}{3} \par
|
|
\hintcontent{Which piece performed the last capture on a black square?}
|
|
|
|
\vspace{2mm}
|
|
Done: \tab\threestars{1}{3}{0}
|
|
\end{hintlist}
|
|
|
|
|
|
\begin{solution}
|
|
|
|
Which piece performed the last capture on a black square? It couldn't have been a white pawn, which haven't moved.
|
|
It couldn't have been the white king, which is trapped; or the black king, which is restricted to white squares.
|
|
|
|
\vspace{2mm}
|
|
|
|
It must have been the piece we can't see, which therefore stands on a black square.
|
|
|
|
\end{solution}
|
|
|
|
\vfill
|
|
\pagebreak
|
|
|
|
|
|
% Sherlock, another monochromatic
|
|
\problem{The knight's grave}
|
|
%\difficulty{3}{5}
|
|
\onestars{4}
|
|
In the game below, no pieces have moved from a black square to a white square, or from a white square to a black square.
|
|
The white king has made less than fourteen moves. \par
|
|
Use this information to show that a pawn was promoted. \par
|
|
|
|
\manyboards{
|
|
ke8,
|
|
Pb2,Pd2,
|
|
Ke1
|
|
}
|
|
|
|
\begin{hintlist}
|
|
Part 1: \tab\threestars{0}{1}{3} \par
|
|
\hintcontent{
|
|
Who took the knights? Only one of them is interesting---most are easy to account for.
|
|
}
|
|
\vspace{2mm}
|
|
Done: \tab\threestars{1}{3}{0}
|
|
\end{hintlist}
|
|
|
|
\begin{solution}
|
|
Knights always move to a different colored square, so all four missing knights must have been captured on their home square.
|
|
What pieces captured them?
|
|
|
|
\vspace{2mm}
|
|
|
|
We can easily account for the white knights and the black knight on G8, but who could've captured the knight from B8?
|
|
The only white pieces that can move to black squares are pawns, the Bishop (which is trapped on C1), the rook (which is stuck on column A and row 1), or the king (which would need at least 14 moves to do so).
|
|
|
|
\vspace{2mm}
|
|
|
|
If this knight was captured by a pawn, that pawn would be immediately promoted. If it was captured by a piece that wasn't a pawn, that piece must be a promoted pawn.
|
|
\end{solution}
|
|
|
|
\vfill
|
|
\pagebreak
|
|
|
|
|
|
|
|
|
|
|
|
|
|
% Arabian Knights, intro (given with solution)
|
|
\problem{Promotion?}
|
|
%\difficulty{2}{5}
|
|
\onestars{6}
|
|
|
|
It is White's move. Have there been any promotions this game? \par
|
|
|
|
\manyboards{
|
|
Pb2,Pe2,kf2,Pg2,Ph2,
|
|
Bc1,Kd1,Rh1
|
|
}
|
|
|
|
\begin{hintlist}
|
|
Part 1: \tab\threestars{0}{4}{2} \par
|
|
\hintcontent{
|
|
The black king must have moved from F1. (Why not G1?) \par
|
|
This would be impossible if something hadn't blocked check from the white rook.
|
|
}
|
|
|
|
\vspace{2mm}
|
|
Done: \tab\threestars{4}{2}{0}
|
|
\end{hintlist}
|
|
|
|
|
|
\begin{solution}
|
|
|
|
Since it is White's move, Black has just moved his king. Where did he move it from?
|
|
Not E1, E3, F3, or G3, since that implies Black had moved into check before. \par
|
|
|
|
\vspace{2mm}
|
|
|
|
The only remaining possibilities are F1 and G1. \par
|
|
G1 is again impossible: how would the king get there without moving into check? \par
|
|
F1, therefore, is the only choice. If we place the king on F1, we see that another piece must prevent check from the white rook.
|
|
This must have been a white black-square bishop, which moved to F2 to reveal that check, and was then captured by the black king.
|
|
|
|
\vspace{2mm}
|
|
|
|
However, there is already a white black-square bishop on the board! We can get a second only by promoting a pawn, so the answer is \say{yes.}
|
|
|
|
\end{solution}
|
|
|
|
\vfill
|
|
\pagebreak
|
|
|
|
|
|
|
|
% Sherlock Holmes, two bagatelles (1)
|
|
\problem{Whodunit}
|
|
%\difficulty{2}{5}
|
|
\onestars{6}
|
|
|
|
It is Black's move. Can Black castle? \par
|
|
\hint{Remember the rules of chess: you may not castle if you've moved your rook.}
|
|
|
|
\manyboards{
|
|
ra8,bc8,ke8,rh8,
|
|
pa7,pc7,pe7,pg7,
|
|
pb6,pf6,ph6,
|
|
Pa3,
|
|
Pb2,Pc2,Pd2,Pe2,Pf2,Pg2,Ph2,
|
|
Bc1,Qd1,Ke1,Bf1
|
|
}
|
|
|
|
\begin{hintlist}
|
|
Part 1: \tab\threestars{0}{4}{2} \par
|
|
\hintcontent{
|
|
Black captured a knight on his last move. \par
|
|
Why do we know this, and how did he do it?
|
|
}
|
|
|
|
\vspace{2mm}
|
|
Done: \tab\threestars{4}{2}{0}
|
|
\end{hintlist}
|
|
|
|
|
|
\begin{solution}
|
|
White's last move was with the pawn. \par
|
|
Black's last move must have been to capture the white piece which moved before that.
|
|
|
|
\vspace{2mm}
|
|
|
|
This piece would have to have been a knight, since the white rooks could not have got out onto the board.
|
|
It is clear that none of the black pawns captured this knight.
|
|
The black rook on A8 couldn't have captured it either, because there is no square that
|
|
the knight could have moved from to get to that position.
|
|
|
|
\vspace{2mm}
|
|
|
|
The black bishop couldn't have captured the knight either, since the only square the
|
|
knight could have come from is D6, where it would have been checking the king.
|
|
|
|
\vspace{2mm}
|
|
|
|
So, the black king or the rook on H8 made this capture. Therefore, Black can't castle.
|
|
\end{solution}
|
|
|
|
\vfill
|
|
\pagebreak
|
|
|
|
|
|
|
|
|
|
|
|
|
|
|
|
|
|
|
|
|
|
% Sherlock Holmes, two bagatelles (2)
|
|
\problem{Castle contradiction}
|
|
%\difficulty{2}{5}
|
|
\startimes{7}
|
|
|
|
Neither Black nor White captured a piece on their last move. \par
|
|
It is Black's move. Can he castle? \par
|
|
\hint{What was White's last move? Check the cases.}
|
|
|
|
\manyboards{
|
|
ke8,rh8,
|
|
pc4,
|
|
Pf3,
|
|
Pc2,Pf2,Pg2,
|
|
bd1,Rf1,Kg1
|
|
}
|
|
|
|
|
|
\begin{hintlist}
|
|
Part 1: \tab\threestars{0}{4}{3} \par
|
|
\hintcontent{
|
|
If White moved his king, Black cannot castle. Why? \par
|
|
That's the simple case. The other option: White castled. What did Black do before that?
|
|
(Also, there was a promotion in this case.)
|
|
}
|
|
|
|
\vspace{2mm}
|
|
Done: \tab\threestars{4}{3}{0}
|
|
\end{hintlist}
|
|
|
|
|
|
\begin{solution}
|
|
If White's last move was with the king, then the black rook moved to check him and Black can't castle.
|
|
|
|
\vspace{2mm}
|
|
|
|
If White's last move wasn't with the king, White must have castled. \par
|
|
What was Black's last move? \par
|
|
If it was with the king or rook, Black can't castle.
|
|
|
|
\vspace{2mm}
|
|
|
|
It could not have been with the bishop, since then White would have had no move immediately before that.
|
|
Now, suppose Black moved his pawn. Then White's preceding move must have been with the pawn from E2,
|
|
capturing a piece on F3. This means that the bishop on D1 is a promoted bishop. The promoting pawn must
|
|
have come from D7, passed D2, checked the white king, making it move!
|
|
This contradicts our assumption that White has just castled.
|
|
\end{solution}
|
|
|
|
\vfill
|
|
\pagebreak
|
|
|
|
|
|
|
|
|
|
|
|
|
|
|
|
|
|
|
|
|
|
|
|
|
|
% Arabian Knights, intro (given with solution)
|
|
\problem{A matter of order}
|
|
%\difficulty{3}{5}
|
|
\startimes{8}
|
|
|
|
A black bishop captured a White piece earlier in this game. \par
|
|
Which bishop was it, and what did it capture? \par
|
|
\hint{Black and White start with 16 pieces each.}
|
|
|
|
\manyboards{
|
|
ra8,qd8,ke8,
|
|
pa7,pc7,pd7,pf7,ph7,
|
|
pb6,nc6,pe6,nf6,ph6,
|
|
Bb5,be5,
|
|
Pe4,bg4,
|
|
Pc3,Nf3,
|
|
Pa2,Pb2,Pc2,Qe2,Pf2,Pg2,Ph2,
|
|
Kc1,Rd1,Rh1
|
|
}
|
|
|
|
|
|
\begin{hintlist}
|
|
Part 1: \tab\threestars{0}{1}{7} \par
|
|
\hintcontent{
|
|
\begin{itemize}
|
|
\item How many pieces are missing? Where were the missing ones captured?
|
|
\item How did those pieces get to the place they were captured?
|
|
\end{itemize}
|
|
}
|
|
|
|
Part 2: \tab\threestars{1}{2}{5} \par
|
|
\hintcontent{
|
|
\begin{itemize}
|
|
\item The pawn on C3 came from D2, capturing a black rook.
|
|
\item The black rook it captured couldn't have moved there before...
|
|
\end{itemize}
|
|
}
|
|
|
|
Part 3: \tab\threestars{3}{3}{2} \par
|
|
\hintcontent{
|
|
\begin{itemize}
|
|
\item ...the black pawn on G7 captured a white piece on H6.
|
|
\item What else is missing?
|
|
\end{itemize}
|
|
}
|
|
|
|
\vspace{2mm}
|
|
Done: \tab\threestars{6}{2}{0}
|
|
\end{hintlist}
|
|
|
|
|
|
\begin{solution}
|
|
First, notice that the pawn on C3 came from D2 by capturing a piece. \par
|
|
This must have been a black rook, which is the only missing black piece.
|
|
|
|
\vspace{2mm}
|
|
|
|
This black rook couldn't have moved there before the black pawn on G7 captured a white piece on H6.
|
|
This piece couldn't have been the missing white bishop, because that bishop would still be trapped by the pawn on D2.
|
|
Therefore, the missing white knight was captured on H6.
|
|
|
|
\vspace{2mm}
|
|
|
|
The only other missing white piece is the black-square bishop, which must have been captured by the black bishop on E5.
|
|
|
|
\end{solution}
|
|
|
|
\vfill
|
|
\pagebreak |